Difference between revisions of "1989 AIME Problems/Problem 13"

m (Solution)
(Solution)
Line 5: Line 5:
 
We first show that we can choose at most 5 numbers from <math>\{1, 2, \ldots , 11\}</math> such that no two numbers have a difference of <math>4</math> or <math>7</math>. We take the smallest number to be <math>1</math>, which rules out <math>5,8</math>. Now we can take at most one from each of the pairs: <math>[2,9]</math>, <math>[3,7]</math>, <math>[4,11]</math>, <math>[6,10]</math>. Now, <math>1989 = 180\cdot 11 + 9</math>. Because this isn't an exact multiple of <math>11</math>, we need to consider some numbers separately.  
 
We first show that we can choose at most 5 numbers from <math>\{1, 2, \ldots , 11\}</math> such that no two numbers have a difference of <math>4</math> or <math>7</math>. We take the smallest number to be <math>1</math>, which rules out <math>5,8</math>. Now we can take at most one from each of the pairs: <math>[2,9]</math>, <math>[3,7]</math>, <math>[4,11]</math>, <math>[6,10]</math>. Now, <math>1989 = 180\cdot 11 + 9</math>. Because this isn't an exact multiple of <math>11</math>, we need to consider some numbers separately.  
  
Notice that <math>1969 = 180\cdot11 - 11 = 179\cdot11</math>. Therefore we can put the last <math>1969</math> numbers into groups of 11. Now let's examine <math>\{1, 2, \ldots , 20\}</math>. If we pick <math>1, 3, 4, 6, 9</math> from the first <math>11</math> numbers, then we're allowed to pick <math>11 + 1</math>, <math>11 + 3</math>, <math>11 + 4</math>, <math>11 + 6</math>, <math>11 + 9</math>. This means we get 10 members from the 20 numbers. Our answer is thus <math>179\cdot 5 + 10 = \boxed{905}</math>.
+
Notice that <math>1969 = 180\cdot11 - 11 = 179\cdot11</math> (*). Therefore we can put the last <math>1969</math> numbers into groups of 11. Now let's examine <math>\{1, 2, \ldots , 20\}</math>. If we pick <math>1, 3, 4, 6, 9</math> from the first <math>11</math> numbers, then we're allowed to pick <math>11 + 1</math>, <math>11 + 3</math>, <math>11 + 4</math>, <math>11 + 6</math>, <math>11 + 9</math>. This means we get 10 members from the 20 numbers. Our answer is thus <math>179\cdot 5 + 10 = \boxed{905}</math>.
 +
 
 +
Remarks
 +
(*) Suppose that you choose the values 1, 2, 4, 7, and 10. Because <math>1989 = 180 \times 11 + 9</math>, this is not maximized. It is only maximized if we include the last element of the final set of 11, which is 10 (this is <math>\text{mod}(11)</math> btw). To include the final element, we have to rewrite 1989 as <math>179 \times 11 + 20</math>, which includes the final element and increases our set by 1 element.
  
 
== See also ==
 
== See also ==

Revision as of 04:07, 17 December 2023

Problem

Let $S$ be a subset of $\{1,2,3,\ldots,1989\}$ such that no two members of $S$ differ by $4$ or $7$. What is the largest number of elements $S$ can have?

Solution

We first show that we can choose at most 5 numbers from $\{1, 2, \ldots , 11\}$ such that no two numbers have a difference of $4$ or $7$. We take the smallest number to be $1$, which rules out $5,8$. Now we can take at most one from each of the pairs: $[2,9]$, $[3,7]$, $[4,11]$, $[6,10]$. Now, $1989 = 180\cdot 11 + 9$. Because this isn't an exact multiple of $11$, we need to consider some numbers separately.

Notice that $1969 = 180\cdot11 - 11 = 179\cdot11$ (*). Therefore we can put the last $1969$ numbers into groups of 11. Now let's examine $\{1, 2, \ldots , 20\}$. If we pick $1, 3, 4, 6, 9$ from the first $11$ numbers, then we're allowed to pick $11 + 1$, $11 + 3$, $11 + 4$, $11 + 6$, $11 + 9$. This means we get 10 members from the 20 numbers. Our answer is thus $179\cdot 5 + 10 = \boxed{905}$.

Remarks (*) Suppose that you choose the values 1, 2, 4, 7, and 10. Because $1989 = 180 \times 11 + 9$, this is not maximized. It is only maximized if we include the last element of the final set of 11, which is 10 (this is $\text{mod}(11)$ btw). To include the final element, we have to rewrite 1989 as $179 \times 11 + 20$, which includes the final element and increases our set by 1 element.

See also

1989 AIME (ProblemsAnswer KeyResources)
Preceded by
Problem 12
Followed by
Problem 14
1 2 3 4 5 6 7 8 9 10 11 12 13 14 15
All AIME Problems and Solutions

The problems on this page are copyrighted by the Mathematical Association of America's American Mathematics Competitions. AMC logo.png